¿Por qué los campos magnéticos son una consecuencia de la relatividad especial? [duplicar]

Estoy muy confundido acerca de por qué es una consecuencia de la relatividad especial.

Respuestas (3)

Olvidémonos de cualquier cosa cuantitativa en absoluto.

La relatividad especial te da la contracción de la longitud, por lo que, cuando te mueves a cierta velocidad, las distancias a lo largo de tu dirección de movimiento se comprimen. Entre muchas otras cosas, esto significa que los volúmenes se reducirán, lo que también significa que las densidades aumentarán.

Ahora, el electromagnetismo nos dice que la fuerza eléctrica es proporcional a la densidad de carga. Entonces, ingenuamente, esperaríamos que la fuerza eléctrica en una partícula de prueba externa a la distribución de carga fuera mayor en un marco de referencia reforzado. Esto, sin embargo, contradice la suposición central de la relatividad especial de que la fuerza neta sobre un objeto no depende de la velocidad del marco de referencia.

Por lo tanto, necesita alguna fuerza nueva que no esté presente en el marco de referencia estacionario. Bueno, en el marco reforzado, las cargas comprimidas se mueven, por lo que hay una corriente, por lo que tal vez podría cancelar su exceso de fuerza con alguna fuerza que dependa de la distribución actual del espacio-tiempo. Si resuelve esto, resulta que el magentismo hace exactamente el truco, y si tiene en cuenta tanto la electricidad como el magnetismo, entonces la fuerza neta sobre la partícula no depende de si está en un marco de referencia estacionario o en movimiento.

Jerry, está bien si podemos explicar la inducción de campos magnéticos por la aceleración de partículas con carga eléctrica. Pero, ¿no sería una simplificación no mencionar que estos electrones llevan un campo magnético intrínseco y no sería mejor mencionar que este campo existe sin cambios y notable en todos los marcos de referencia?
No estoy de acuerdo. Las cargas en una barra cargada aún en pie ejercen una fuerza de Coulomb sobre una carga puntual aún en pie junto a ella. Cuando la barra comienza a moverse, se contrae y la carga puntual siente una fuerza mayor, porque algunas cargas en la barra se acercaron cuando la barra se contrajo. Fuerza aumentada. Sin magnetismo. El impulso es casi lo mismo. Si entendí mal, entonces la respuesta debería ser más clara.
@stuffu A la fuerza TIENE que no importarle si la distribución de carga es estacionaria o se mueve a una velocidad constante, eso es solo un axioma de la relatividad especial. Si la fuerza fuera mayor en el cuadro potenciado, podría medir la fuerza mayor y saber que fue potenciado, después de todo. La razón es que tiene una fuerza magnética en el marco potenciado y no tiene ninguna en el marco no potenciado, y la fuerza magnética cancela el exceso de fuerza eléctrica.
@HolgerFiedler: esta respuesta no tiene nada que ver con las cargas eléctricas fundamentales y los momentos magnéticos. No se trata de "de dónde viene la fuerza magnética", sino de "Si tienes relatividad especial y electricidad, entonces necesariamente TENDRÁS que tener magnetismo".
@JerrySchirmer No, en serio. Cuando ingresa a un marco potenciado desde un marco donde las cargas estaban inmóviles, podrá ver cómo se mueven las cargas y podrá medir que la fuerza entre las cargas es menor que la que había en el marco anterior. Las cargas miden una fuerza entre ellas, tú mides una fuerza diferente entre ellas. (¿Tal vez sea difícil de creer?) Cuantitativamente: las cargas miden una fuerza de Coulomb, tú mides una fuerza de Coulomb menos una fuerza magnética.
@stuffu: no estoy hablando de fuerzas internas en la distribución de carga (¿dónde hablo de fuerzas internas en una distribución de carga?). Estoy hablando de la fuerza que siente una partícula de prueba que se mueve a través del campo establecido por la distribución de carga.
Si bien no discuto que bajo la relatividad especial una teoría con fuerza electrostática también requiere una fuerza magnética para obedecer la invariancia relativista, siento que la explicación aquí es demasiado simple para ser completamente convincente. Todo lo que se dice aquí debería aplicarse a la gravedad de la misma manera que a la fuerza electrostática, después de todo ambas son 1 / r 2 fuerzas generadas por una densidad. Entonces, ¿por qué la gravedad no necesita también un campo gravito-magnético que la acompañe?
@JerrySchirmer Entonces, ¿qué es esto de que algo no nota ninguna diferencia debido al movimiento? La partícula de prueba nota una diferencia. Partícula de prueba moviéndose a través de la distribución de carga: "La distribución de carga se contrae, por eso hay una gran fuerza entre la distribución de carga y yo". Distribución de carga: "El campo de la partícula de prueba se contrae, por eso es más fuerte en algunos lugares, más débil en otros lugares"
@stuffu: la fuerza es la misma en ambos marcos. los q v × B del campo magnético cancela el campo eléctrico impulsado debido a la distribución de carga más densa. Búscalo en Griffiths. Esto es solo un hecho, y su intuición aquí es incorrecta.
@MarcvanLeeuwen: Mi objetivo aquí no es una prueba definitiva, es un argumento motivador. Por supuesto, puede respaldar esto con un cálculo. Además, es ciertamente cierto que obtienes efectos como este en la gravedad, y de hecho, la gravedad perturbadora tiene un efecto llamado "gravetomagnetismo", ejemplificado por el efecto Lens-Thirring medido por la sonda de gravedad B: en.wikipedia.org/wiki /Lense%E2%80%93Thirring_precession , entre otras cosas
@JerrySchirmer Pruebe la partícula en el cuadro 1 y también en el cuadro 2: "Mi velocidad es 0, por lo tanto, no siento ninguna fuerza de Lorentz".
@stuffu: en el cuadro potenciado, la partícula de prueba tiene una velocidad. Eso es lo que es un impulso.
@JerrySchirmer Ahora, si supiera la velocidad de la distribución de carga en el cuadro reforzado, entonces tal vez conocía el cuadro reforzado lo suficientemente bien como para entenderlo.
@stuffu: todo el local de construcción dos marcos. 1) La distribución es estática y la partícula de prueba es estática 2) un marco reforzado que se mueve a una velocidad v con respecto a éste a lo largo del eje de la línea de carga. El punto es que la densidad de carga aumentará, pero ahora habrá una corriente y la partícula de prueba se moverá con una velocidad v . Dado que la fuerza magnética entre dos corrientes paralelas es atractiva, la fuerza magnética resultante estará en la dirección opuesta a la fuerza eléctrica y, de hecho, compensará exactamente el aumento de la fuerza eléctrica.

Para resumir rápidamente de una manera diferente algunas de las ideas presentadas en el primer enlace dado por ACuriousMind: no , el campo magnético no se deriva de la electrostática + relatividad especial.

Hay una forma muy sencilla de asegurarse de ello. En el ejemplo estándar de Purcell, tiene un marco en el que un cable eléctricamente neutro que transporta una corriente estacionaria yo genera un campo magnético constante B . En esta configuración solo hay un campo magnetostático. B y cero campo electrico mi = 0 .

En este marco, una carga libre se mueve paralela al alambre con velocidad v y experimentará una fuerza de Lorentz q v × B .

Luego, el argumento estándar continúa aplicando un impulso para colocarse en un nuevo marco de inercia que se mueve junto con la carga móvil y, por lo tanto, tanto la velocidad instantánea de esa carga como la fuerza de Lorentz asociada desaparecen en ese nuevo marco; abracadabra la fuerza magnetica ha desaparecido!

¿Eso implica que el campo magnético no existe en ese marco? No.

Es fácil asegurarse de eso porque:

  • a) todavía hay una corriente en el cable, por lo que debe haber un campo magnético asociado y,

  • b) la cantidad mi 2 C 2 B 2 es un invariante de Lorentz tal que

    C 2 B 2 = mi 2 C 2 B 2
    donde las cantidades primadas están en el marco de co-movimiento. De la ecuación anterior se deduce que el lado derecho debe ser negativo; lo que significa que tiene que haber un campo magnético en el marco de movimiento conjunto sin importar cuál pueda ser la fuerza en alguna carga móvil seleccionada.

Para demostrar que en algunas circunstancias no hay campo magnético, habría que demostrar que no hay fuerza magnética sobre ninguna carga en movimiento; lo cual no es el caso en absoluto en la "derivación" habitual de los libros de texto.

Ahora, para ser justos, la fuerza magnética (o más bien su expresión) desaparece cuando uno se coloca en un marco que se mueve junto con una carga móvil, pero eso se debe a que la fuerza electromagnética es un vector de 4 y aplicarle un impulso simplemente da Nosotros una relación entre la fuerza observada en un marco (solo la fuerza de Lorentz) y la fuerza observada en el marco co-móvil (electrostática pura). Tenga en cuenta que esto sería solo una relación y no se puede inferir de ella que la electrostática es de alguna manera más fundamental que el magnetismo, ya que eso daría preferencia a los marcos en los que las fuerzas sobre algunas partículas se expresan solo en términos de fuerzas eléctricas; lo que en sí mismo contradice toda la historia sobre el principio de la relatividad.

Creo que el argumento más limpio aquí es que "si solo tienes fuerzas eléctricas y la invariancia de Lorentz, debes terminar con fuerzas magnéticas". No es un argumento que uno sea más "fundamental" que el otro, tanto como se necesita electricidad y mangetismo para tener una teoría consistente.
@JerrySchirmer: claro, pero tenga en cuenta que la pregunta original se trata de campos magnéticos, no de fuerzas magnéticas. Y nuevamente, la razón es porque uno puede pensar en el nivel de 4 vectores cuando se trata de componentes de fuerza, por lo que, por supuesto, no se captura completamente la imagen completa de los campos, ya que centrarse en 1 partícula no dice nada sobre todas las otras posibles cargas en movimiento.
Claro, pero una fuerza magnética presupone un campo magnético y viceversa. La forma más obvia de conocer un campo es medir fuerzas en una partícula de prueba, al menos. Pero claro, entiendo el punto de su respuesta, e incluso he votado a favor.

Si tenemos una carga de prueba en movimiento y queremos saber qué fuerza la está acelerando, una forma sencilla de calcularlo es esta: primero transformamos a un marco donde la velocidad de la carga es cero. La relatividad especial nos dice cómo nos transformamos correctamente a ese marco.

Luego, en este nuevo marco, cada carga ejerce una fuerza de Coulomb sobre la carga de prueba. Calculamos la fuerza neta que siente la carga de prueba de todas las demás cargas, llamemos a esa fuerza F.

Finalmente, descubrimos cómo se ve esa fuerza F en el marco original, al transformar a ese marco, nuevamente la relatividad especial nos dice cómo se hace eso. Como nos interesa la fuerza, transformamos la fuerza.

Ahora bien, si hay una persona que dice que un campo magnético está causando una fuerza sobre esa partícula de prueba, entonces el campo magnético debe ser tal que al calcular la fuerza sobre la partícula, el resultado sea el mismo que calculamos usando la relatividad especial. Entonces, la relatividad especial dicta cómo es el campo magnético.

Un caso especial que quiero mencionar: una carga de prueba y otra carga se mueven una al lado de la otra a la misma velocidad. Cuando transformamos al marco de la carga de prueba, calculamos que la fuerza que afecta a la carga de prueba es F. Cuando volvemos a transformar al marco original, aprendemos que la fuerza que afecta a la carga de prueba en este marco es F/gamma, donde gamma es la misma gamma que en la transformación de longitud relativista, o transformación de tiempo relativista.